“Not advocated”
Does not advocated here mean the author of passage b disagrees with the policy, or does it mean s...
kassidee on June 27, 2020
  • November 2018 LSAT
  • SEC3
  • Q19
1
Reply
help please
what signals words were used for us to infer that the author was being sympathetic.
YulissaCardoza on June 26, 2020
  • November 2018 LSAT
  • SEC3
  • Q22
3
Replies
Why D?
Why would D be correct here? Thank you! (Nov 2018 LSAT #24)
samlhoover on June 25, 2020
  • November 2018 LSAT
  • SEC3
  • Q24
3
Replies
Why D?
Hello! Do we know D is correct because the first paragraph explains what typifies the classica...
Lucie on May 4, 2020
  • November 2018 LSAT
  • SEC3
  • Q11
2
Replies
C
"The musical's conventions, Bordwell argues, cue viewers . to expect a different structure—alte...
tomgbean on January 21, 2020
  • November 2018 LSAT
  • SEC3
  • Q13
1
Reply
"Realistic"
I am unsure how to select B as the correct answer, because it appears that multiple definitions o...
Lena-Chan on January 8, 2020
  • November 2018 LSAT
  • SEC3
  • Q9
1
Reply
Explanation
I don't understand why this is C. Can I get a breakdown?
mluna on December 28, 2019
  • November 2018 LSAT
  • SEC3
  • Q5
2
Replies
Most Likely Agree
In these type of questions, will the correct answer be a restatement from the passage?
mluna on December 2, 2019
  • November 2018 LSAT
  • SEC3
  • Q6
3
Replies
Explanation of Answer Choices
Can you please explain why it is not the fifth paragraph? I understood it as the third paragraph ...
Aley on November 25, 2019
  • November 2018 LSAT
  • SEC3
  • Q23
1
Reply
please explain?
please explain why correct answer is A?
maonuo on November 16, 2019
  • November 2018 LSAT
  • SEC3
  • Q15
1
Reply
Why is B correct?
Hello! Could you explain more about the stances between the passages, and in particular the la...
Lucie on November 12, 2019
  • November 2018 LSAT
  • SEC3
  • Q17
1
Reply
How do we eliminate C?
Is C in correct because it does not say the evidence of drought occurred during the Bronze Age?
#JW on November 6, 2019
  • November 2018 LSAT
  • SEC3
  • Q4
2
Replies
Explanation
Can someone explain this?
mluna on November 5, 2019
  • November 2018 LSAT
  • SEC3
  • Q16
1
Reply
The correct answer E is not shown for some tech...
Please update.
samxinghaoli on June 29, 2019
  • November 2018 LSAT
  • SEC3
  • Q19
2
Replies